Fundamentals of Nursing

Lakukan tugas rumah & ujian kamu dengan baik sekarang menggunakan Quizwiz!

The nurse is teaching an obese client, newly diagnosed with arteriosclerosis, about reducing the risk of a heart attack or stroke. Which health promotion brochure is most important for the nurse to provide to this client? A. "Monitoring Your Blood Pressure at Home" B. "Smoking Cessation as a Lifelong Commitment" C. "Decreasing Cholesterol Levels Through Diet" D. "Stress Management for a Healthier You"

C. "Decreasing Cholesterol Levels Through Diet" Rationale: A health promotion brochure about decreasing cholesterol (C) is most important to provide this client, because the most significant risk factor contributing to development of arteriosclerosis is excess dietary fat, particularly saturated fat and cholesterol. (A) does not address the underlying causes of arteriosclerosis. (B and D) are also important factors for reversing arteriosclerosis but are not as important as lowering cholesterol (C).

A client diagnosed with primary open-angle glaucoma received a prescription for miotic eye drops, pilocarpine HCl (Pilocarpine). What instructions should the nurse plan to include in this client's teaching? A. "Administer the medication directly on the cornea." B. "Wash your hands after each administration of eye drops." C. "Do not allow the dropper bottle to touch the eye." D. "Squeeze your eye closed after administering the drops."

C. "Do not allow the dropper bottle to touch the eye."

One week after being told that she has terminal cancer with a life expectancy of 3 weeks, a female client tells the nurse, "I think I will plan a big party for all my friends." How should the nurse respond? A. "You may not have enough energy before long to hold a big party." B. "Do you mean to say that you want to plan your funeral and wake?" C. "Planning a party and thinking about all your friends sounds like fun." D. "You should be thinking about spending your last days with your family."

C. "Planning a party and thinking about all your friends sounds like fun." Rationale: Setting goals that bring pleasure are appropriate and should be encouraged by the nurse (C) as long as the nurse does not perpetuate a client's denial. (A) is a negative response, implying that the client should not plan a party. (B) puts words in the client's mouth that may not be accurate. The nurse should support the client's goals rather than telling the client how to spend her time (D).

An older adult who recently began self-administration of insulin calls the nurse daily to review the steps that should be taken when giving an injection. The nurse has assessed the client's skills during two previous office visits and knows that the client is capable of giving the daily injection. Which response by the nurse is likely to be most helpful in encouraging the client to assume total responsibility for the daily injections? A. "I know you are capable of giving yourself the insulin." B. "Giving yourself the injection seems to make you nervous." C. "When I watched you give yourself the injection, you did it correctly." D. "Tell me what you want me to do to help you give yourself the injection at home."

C. "When I watched you give yourself the injection, you did it correctly." Rationale: The nurse needs to focus on the client's positive behaviors, so focusing on the client's demonstrated ability to self-administer the injection (C) is likely to reinforce his level of competence without sounding punitive. (A) does not focus on the specific behaviors related to giving the injection and could be interpreted as punitive. (B) uses reflective dialogue to assess the client's feelings, but telling the client that he is nervous may serve as a negative reinforcement of this behavior. (D) reinforces the client's dependence on the nurse.

A community hospital is opening a mental health services department. Which document should the nurse use to develop the unit's nursing guidelines? A. Americans with Disability Act of 1990 B. ANA Code of Ethics with Interpretative Statements C. ANA's Scope and Standards of Nursing Practice D. Patient's Bill of Rights of 1990

C. ANA's Scope and Standards of Nursing Practice Rationale: The ANA Scope of Standards of Practice for Psychiatric-Mental Health Nursing (C) serves to direct the philosophy and standards of psychiatric nursing practice. (A and D) define the client's rights. (B) provides ethical guidelines for nursing.

The nurse is preparing to administer 10 mL of liquid potassium chloride (Kay Ciel) through a feeding tube, followed by 10 mL of liquid acetaminophen (Tylenol). Which action should the nurse include in this procedure? A. Dilute each of the medications with sterile water prior to administration. B. Mix the medications in one syringe before opening the feeding tube. C. Administer water between the doses of the two liquid medications. D. Withdraw any fluid from the tube before instilling each medication.

C. Administer water between the doses of the two liquid medications. Rationale: Water should be instilled into the feeding tube between administering the two medications (C) to maintain the patency of the feeding tube and ensure that the total dose of medication enters the stomach and does not remain in the tube. These liquid medications do not need to be diluted (A) when administered via a feeding tube and should be administered separately (B), with water instilled between each medication (D).

A client in a long-term care facility reports to the nurse that he has not had a bowel movement in 2 days. Which intervention should the nurse implement first? A. Instruct the caregiver to offer a glass of warm prune juice at mealtimes. B. Notify the health care provider and request a prescription for a large- volume enema. C. Assess the client's medical record to determine the client's normal bowel pattern. D. Instruct the caregiver to increase the client's fluids to five 8-ounce glasses per day.

C. Assess the client's medical record to determine the client's normal bowel pattern. Rationale: This client may not routinely have a daily bowel movement, so the nurse should first assess this client's normal bowel habits before attempting any intervention

After the nurse tells an older client that an IV line needs to be inserted, the client becomes very apprehensive, loudly verbalizing a dislike for all health care providers and nurses. How should the nurse respond? A. Ask the client to remain quiet so the procedure can be performed safely. B. Concentrate on completing the insertion as efficiently as possible. C. Calmly reassure the client that the discomfort will be temporary. D. Tell the client a joke as a means of distraction from the procedure.

C. Calmly reassure the client that the discomfort will be temporary. Rationale: The nurse should respond with a calm demeanor (C) to help reduce the client's apprehension. After responding calmly to the client's apprehension, the nurse may implement (A, B, or D) to ensure safe completion of the procedure.

A female client with frequent urinary tract infections (UTIs) asks the nurse to explain her friend's advice about drinking a glass of juice daily to prevent future UTIs. Which response is best for the nurse? A. Orange juice has vitamin C that deters bacterial growth. B. Apple juice is the most useful in acidifying the urine. C. Cranberry juice stops pathogens' adherence to the bladder. D. Grapefruit juice increases absorption of most antibiotics.

C. Cranberry juice stops pathogens' adherence to the bladder. Rationale: Cranberry juice (C) maintains urinary tract health by reducing the adherence of Escherichia coli bacteria to cells within the bladder. (A, B, and D) have not been shown to be as effective as cranberry juice (C) in preventing UTIs.

When taking a client's blood pressure, the nurse is unable to distinguish the point at which the first sound was heard. Which is the best action for the nurse to take? A. Deflate the cuff completely and immediately reattempt the reading. B. Reinflate the cuff completely and leave it inflated for 90 to 110 seconds before taking the second reading. C. Deflate the cuff to zero and wait 30 to 60 seconds before reattempting the reading. D. Document the exact level visualized on the sphygmomanometer where the first fluctuation was seen.

C. Deflate the cuff to zero and wait 30 to 60 seconds before reattempting the reading. Rationale: Deflating the cuff for 30 to 60 seconds (C) allows blood flow to return to the extremity so that an accurate reading can be obtained on that extremity a second time. (A) could result in a falsely high reading. (B) reduces circulation, causes pain, and could alter the reading. (D) is not an accurate method of assessing blood pressure.

The nurse who is preparing to give an adolescent client a prescribed antipsychotic medication notes that parental consent has not been obtained. Which action should the nurse take? A. Review the chart for a signed consent for hospitalization. B. Get the health care provider's permission to give the medication. C. Do not give the medication and document the reason. D. Complete an incident report and notify the parents

C. Do not give the medication and document the reason. Rationale: The nurse should not give the medication and should document the reason (C) because the client is a minor and needs a guardian's permission to receive medications. Permission to give medications is not granted by a signed hospital consent (A) or a health care provider's permission (B), unless conditions are met to justify coerced treatment. (D) is not necessary unless the medication had previously been administered.

Which instruction is most important for the nurse to include when teaching a client with limited mobility strategies to prevent venous thrombosis? A. Perform cough and deep breathing exercises hourly. B. Turn from side to side in bed at least every 2 hours. C. Dorsiflex and plantarflex the feet 10 times each hour. D. Drink approximately 4 ounces of water every hour.

C. Dorsiflex and plantarflex the feet 10 times each hour. Rationale: To reduce the risk of venous thrombosis, the nurse should instruct the client in measures that promote venous return, such as dorsiflexion and plantar flexion (C). (A, B, and D) are helpful to prevent other complications of immobility but are less effective in preventing venous thrombus formation than (C).

The nurse has removed the barbiturate capsule from the unit dose wrapper to administer to a male client. The client decides he wants to watch a television program and requests not to take the medication. Which action should the nurse implement? A. Credit the medication back and put in the client's medication box B. Keep the medication and see if the client will want to take it later. C. Have another nurse watch disposal of the medication into disposal container D. Explain that since the medication is a controlled substance it must be taken.

C. Have another nurse watch disposal of the medication into disposal container

In completing a client's preoperative routine, the nurse finds that the operative permit is not signed. The client begins to ask more questions about the surgical procedure. Which action should the nurse take next? A. Witness the client's signature to the permit. B. Answer the client's questions about the surgery. C. Inform the surgeon that the operative permit is not signed and the client has questions about the surgery. D. Reassure the client that the surgeon will answer any questions before the anesthesia is administered.

C. Inform the surgeon that the operative permit is not signed and the client has questions about the surgery. Rationale: The surgeon should be informed immediately that the permit is not signed (C). It is the surgeon's responsibility to explain the procedure to the cliesxnt and obtain the client's signature on the permit. Although the nurse can witness an operative permit (A), the procedure must first be explained by the health care provider or surgeon, including answering the client's questions (B). The client's questions should be addressed before the permit is signed (D).

Urinary catheterization is prescribed for a postoperative female client who has been unable to void for 8 hours. The nurse inserts the catheter, but no urine is seen in the tubing. Which action will the nurse take next? A. Clamp the catheter and recheck it in 60 minutes. B. Pull the catheter back 3 inches and redirect upward. C. Leave the catheter in place and reattempt with another catheter. D. Notify the health care provider of a possible obstruction.

C. Leave the catheter in place and reattempt with another catheter. Rationale: It is likely that the first catheter is in the vagina, rather than the bladder. Leaving the first catheter in place will help locate the meatus when attempting the second catheterization (C). The client should have at least 240 mL of urine after 8 hours. (A) does not resolve the problem. (B) will not change the location of the catheter unless it is completely removed, in which case a new catheter must be used. There is no evidence of a urinary tract obstruction if the catheter could be easily inserted (D).

When evaluating the effectiveness of a client's nursing care, the nurse first reviews the expected outcomes identified in the plan of care. What action should the nurse take next? A. Determine if the expected outcomes were realistic B. Modify the nursing interventions to achieve the client's goals C. Obtain current client data to compare with expected outcomes D. Review related professional standards of care.

C. Obtain current client data to compare with expected outcomes

The nurse is evaluating the fluid balance of the client who was admitted yesterday with dehydration and who has been receiving iv fluids since admission. An increase in which parameter indicates to the nurse that the client is rehydrating. A. Serum haematocrit. B. Urine specific gravity. C. Pulse Rate. D. Urinary output.

C. Pulse Rate.

The nurse observes that a male client on a clear liquid diet has a cup of coffee on his breakfast tray. What action should the nurse implement? A. Consult with the dietician to learn if the client is allowed to drink coffee B. Determine which member of the nursing staff brought the cup of coffee to the client C. Remind the client that no milk, or creamer can be added to the coffee. D. Remove the coffee from the tray, advising the client that it is not included in the diet.

C. Remind the client that no milk, or creamer can be added to the coffee.

When bathing an uncircumcised boy older than 3 years, which action should the nurse take? A. Remind the child to clean his genital area. B. Defer perineal care because of the child's age. C. Retract the foreskin gently to cleanse the penis. D. Ask the parents why the child is not circumcised.

C. Retract the foreskin gently to cleanse the penis. Rationale:The foreskin (prepuce) of the penis should be gently retracted to cleanse all areas that could harbor bacteria (C). The child's cognitive development may not be at the level at which (A) would be effective. Perineal care needs to be provided daily regardless of the client's age (B). (D) is not indicated and may be perceived as intrusive.

A nurse stops at a motor vehicle collision site to render aid until the emergency personnel arrive and applies pressure to a groin wound that is bleeding profusely. Later the client has to have the leg amputated and sues the nurse for malpractice. Which is the most likely outcome of this lawsuit? A. The Patient's Bill of Rights protects clients from malicious intents, so the nurse could lose the case. B. The lawsuit may be settled out of court, but the nurse's license is likely to be revoked. C. There will be no judgment against the nurse, whose actions were protected under the Good Samaritan Act. D. The client will win because the four elements of negligence (duty, breach, causation, and damages) can be proved.

C. There will be no judgment against the nurse, whose actions were protected under the Good Samaritan Act. Rationale: The Good Samaritan Act (C) protects health care professionals who practice in good faith and provide reasonable care from malpractice claims, regardless of the client outcome. Although the Patient's Bill of Rights protects clients, this nurse is protected by the Good Samaritan Act (A). The state Board of Nursing has no reason to revoke a registered nurse's license (B) unless there was evidence that actions taken in the emergency were not done in good faith or that reasonable care was not provided. All four elements of malpractice were not shown (D).

The home health nurse visits a client who has a serum sodium level of 123 mEq/L. To explore possible etiologies for this value, what question should the nurse ask this client? A: "How frequently do you eat processed or canned foods?" B: "Do you drink milk or eat dairy products at each meal?" C: "How much water and ice chips do you have each day?" D: "What amount of your daily meals contains fresh vegetables?"

C: "How much water and ice chips do you have each day?"

Which technique is most important for the nurse to implement when performing a physical assessment? A: A head-to-toe approach. B: The medical systems approach. C: A consistent, systematic approach. D: An approach related to a nursing model.

C: A consistent, systematic approach

The nurse is preparing to feed a newly administered elderly male client who is debilitated but is able to respond to most commands. Before starting to feed the client, which information is most important for the nurse to obtain? A: Client's respiratory rate and lung sounds. B: Prescribed diet. C: Client's ability to chew and swallow. D: Current medications.

C: Client's ability to chew and swallow.

A female client informs the nurse that she uses herbal therapies to supplement her diet and manage common ailments. What information should the nurse offer the client about general use of herbal supplements? A: Most herbs are toxic or carcinogenic and should be used only when proven effective. B: There is no evidence that herbs are safe or effective as compared to conventional supplements in maintaining health. C: Herbs should be obtained from manufacturers with a history of quality control of their supplements. D: Herbal therapies may mask the symptoms of serious disease, so frequent medical evaluation is required during use.

C: Herbs should be obtained from manufacturers with a history of quality control of their supplements.

A nurse is discharging a patient who has been hospitalized for the last 5 days with pneumonia. While providing discharge instructions, the client is noticeably anxious. What action is the most important for the nurse to implement? A: Encourage the patient to take a PRN anxiety medication. B: Include a family member in the teaching session. C: Provide written discharge instructions. D: Instruct the client to repeat the medication discharge plan.

C: Provide written discharge instructions.

A client is admitted with a stage four pressure ulcer that has a black, hardened surface and a light pink wound bed with a malodorous green drainage. Which dressing is best for the nurse to use first? A: Hydrogel. B: Exudate absorber. C: Wet to moist dressing. D: Transparent adhesive film.

C: Wet to moist dressing

A client who has been diagnosed with terminal cancer tells the nurse, "The doctor told me I have cancer and do not have long to live." Which response is best for the nurse to provide? A) "That's correct, you do not have long to live" B) "Would you like me to call your minister?" C) "Don't give up, you still have chemotherapy to try." D) "Yes, your condition is serious."

D) "Yes, your condition is serious."

A middle-aged male client tells the nurse that weeks ago he began exercising four times a week to lose weight and to help him sleep better. He states that it still takes an hour to fall asleep at night. What action should the nurse implement? A) Advice the client that lifestyle changes often take several weeks to be effective. B) Determine the amount of weight the client has lost since increasing his activity. C) Encourage the client to exercise every day to eliminate bedtime wakefulness. D) Ask the client to describe the exercise schedule that he has been following.

D) Ask the client to describe the exercise schedule that he has been following.

The nurse is administering the 0900 medications to a client who was admitted during the night. Which client statement indicates that the nurse should further assess the medication order? A. "At home I take my pills at 8:00 am." B. "It costs a lot of money to buy all of these pills." C. "I get so tired of taking pills every day." D. "This is a new pill I have never taken before.

D. "This is a new pill I have never taken before. Rationale: The client's recognition of a "new" pill requires further assessment (D) to verify that the medication is correct, if it is a new prescription or a different manufacturer, or if the client needs further instruction. The time difference may not be as significant in terms of its effect, but this should be explained (A). Although comments about cost (B) should be considered when developing a discharge plan, (D) is a higher priority. The client's feelings (C) should be acknowledged, but observation of the five rights of medication administration is most essential.

The nurse is teaching a client how to perform progressive muscle relaxation techniques to relieve insomnia. A week later the client reports that he is still unable to sleep, despite following the same routine every night. Which action should the nurse take first? A. Instruct the client to add regular exercise as a daily routine. B. Determine if the client has been keeping a sleep diary. C. Encourage the client to continue the routine until sleep is achieved. D. Ask the client to describe the routine that the client is currently following.

D. Ask the client to describe the routine that the client is currently following. Rationale The nurse should first evaluate whether the client has been adhering to the original instructions (D). A verbal report of the client's routine will provide more specific information than the client's written diary (B). The nurse can then determine which changes need to be made (A). The routine practiced by the client is clearly unsuccessful, so encouragement alone is insufficient (C).

During a routine assessment, an obese 50-year-old female client expresses concern about her sexual relationship with her husband. Which is the best response by the nurse? A. Reassure the client that many obese people have concerns about sex. B. Remind the client that sexual relationships need not be affected by obesity. C. Determine the frequency of sexual intercourse. D. Ask the client to talk about specific concerns.

D. Ask the client to talk about specific concerns. Rationale: (D) provides an opportunity for the client to verbalize her concerns and provides the nurse with more assessment data. (A and B) may not be related to her current concern, assume that obesity is the problem, and are communication blocks. (C) may be appropriate after discussing the concerns she is having.

A client's blood pressure reading is 156/94 mm Hg. Which action should the nurse take first? A. Tell the client that the blood pressure is high and that the reading needs to be verified by another nurse. B. Contact the health care provider to report the reading and obtain a prescription for an antihypertensive medication. C. Replace the cuff with a larger one to ensure an ample fit for the client to increase arm comfort. D. Compare the current reading with the client's previously documented blood pressure readings.

D. Compare the current reading with the client's previously documented blood pressure readings. Rationale: Comparing this reading with previous readings (D) will provide information about what is normal for this client; this action should be taken first. (A) might unnecessarily alarm the client. (B) is premature. Further assessment is needed to determine if the reading is abnormal for this client. (C) could falsely decrease the reading and is not the correct procedure for obtaining a blood pressure reading.

The nurse transcribes the postoperative prescriptions for a client who returns to the unit following surgery and notes that an antihypertensive medication that was prescribed preoperatively is not listed. Which action should the nurse take? A. Consult with the pharmacist about the need to continue the medication. B. Administer the antihypertensive medication as prescribed preoperatively. C. Withhold the medication until the client is fully alert and vital signs are stable. D. Contact the health care provider to renew the prescription for the medication.

D. Contact the health care provider to renew the prescription for the medication. Rationale: Medications prescribed preoperatively must be renewed postoperatively, so the nurse should contact the health care provider if the antihypertensive medication is not included in the postoperative prescriptions (D). The pharmacist (A) does not prescribe medications or renew prescriptions. The nurse must have a current prescriptions before administering any medications (B and C).

During a clinic visit, the mother of a 7-year-old reports to the nurse that her child is often awake until midnight playing and is then very difficult to awaken in the morning for school. Which assessment data should the nurse obtain in response to the mother's report? A. The occurrence of any episodes of sleep apnea B. The child's blood pressure, pulse, and respirations C. Length of rapid eye movement (REM) sleep that the child is experiencing D. Description of the family's home environment

D. Description of the family's home environment Rationale: School-age children often resist bedtime. The nurse should begin by assessing the environment of the home (D) to determine factors that may not be conducive to the establishment of bedtime rituals that promote sleep. (A) often causes daytime fatigue rather than resistance to going to sleep. (B) is unlikely to provide useful data. The nurse cannot determine (C).

The nurse determines that a postoperative client's respiratory rate has increased from 18 to 24 breaths/min. Based on this assessment finding, which intervention is most important for the nurse to implement? A. Encourage the client to increase ambulation in the room. B. Offer the client a high-carbohydrate snack for energy. C. Force fluids to thin the client's pulmonary secretions. D. Determine if pain is causing the client's tachypnea.

D. Determine if pain is causing the client's tachypnea. Rationale: Pain, anxiety, and increasing fluid accumulation in the lungs (D) can cause tachypnea (increased respiratory rate). Encouraging (A) when the respiratory rate is rising above normal limits puts the client at risk for further oxygen desaturation. (B) can increase the client's carbon metabolism, so an alternative source of energy, such as Pulmocare liquid supplement, should be offered instead. (C) could increase respiratory congestion in a client with a poorly functioning cardiopulmonary system, placing the client at risk of fluid overload.

When performing sterile wound care in the acute care setting, the nurse obtains a bottle of normal saline from the bedside table that is labeled "opened" and dated 48 hours prior to the current date. Which is the best action for the nurse to take? A. Use the normal saline solution once more and then discard. B. Obtain a new sterile syringe to draw up the labeled saline solution. C. Use the saline solution and then relabel the bottle with the current date. D. Discard the saline solution and obtain a new unopened bottle.

D. Discard the saline solution and obtain a new unopened bottle. Rationale: Solutions labeled as opened within 24 hours may be used for clean procedures, but only newly opened solutions are considered sterile. This solution is not newly opened and is out of date, so it should be discarded (D). (A, B, and C) describe incorrect procedures.

The mental health nurse plans to discuss a client's depression with the health care provider in the emergency department. There are two clients sitting across from the emergency department desk. Which nursing action is best? A. Only refer to the client by gender. B. Identify the client only by age. C. Avoid using the client's name. D. Discuss the client another time.

D. Discuss the client another time. Rationale: The best nursing action is to discuss the client another time (D). Confidentiality must be observed at all times, so the nurse should not discuss the client when the conversation can be overheard by others. Details can identify the client when referring to the client by gender (A) or age (B), and even when not using the client's name (C).

Which intervention is most important to include in the plan of care for a client at high risk for the development of postoperative thrombus formation? A. Instruct in the use of the incentive spirometer. B. Elevate the head of the bed during all meals. C. Use aseptic technique to change the dressing. D. Encourage frequent ambulation in the hallway.

D. Encourage frequent ambulation in the hallway. Rationale: Thrombus (clot) formation can occur in the lower extremities of immobile clients, so the nurse should plan to encourage activities to increase mobility, such as frequent ambulation (D) in the hallway. (A) helps promote alveolar expansion, reducing the risk for atelectasis. (B) reduces the risk for aspiration. (C) reduces the risk for postoperative infection.

After receiving written and verbal instructions from a clinic nurse about a newly prescribed medication, a client asks the nurse what to do if questions arise about the medication after getting home. How should the nurse respond? A. Provide the client with a list of Internet sites that answer frequently asked questions about medications. B. Advise the client to obtain a current edition of a drug reference book from a local bookstore or library. C. Reassure the client that information about the medication is included in the written instructions. D. Encourage the client to call the clinic nurse or health care provider if any questions arise.

D. Encourage the client to call the clinic nurse or health care provider if any questions arise. Rationale: To ensure safe medication use, the nurse should encourage the client to call the nurse or health care provider (D) if any questions arise. (A, B, and C) may all include useful information, but these sources of information cannot evaluate the nature of the client's questions and the follow-up needed.

The nurse is assisting a client to the bathroom. When the client is 5 feet from the bathroom door, he states, "I feel faint." Before the nurse can get the client to a chair, the client starts to fall. Which is the priority action for the nurse to take? A. Check the client's carotid pulse. B. Encourage the client to get to the toilet. C. In a loud voice, call for help. D. Gently lower the client to the floor.

D. Gently lower the client to the floor. Rationale: (D) is the most prudent intervention and is the priority nursing action to prevent injury to the client and the nurse. Lowering the client to the floor should be done when the client cannot support his own weight. The client should be placed in a bed or chair only when sufficient help is available to prevent injury. (A) is important but should be done after the client is in a safe position. Because the client is not supporting himself, (B) is impractical. (C) is likely to cause chaos on the unit and might alarm the other clients.

Which nursing diagnosis has the highest priority when planning care for a client with an indwelling urinary catheter? A. Self-care deficit B. Functional incontinence C. Fluid volume deficit D. High risk for infection

D. High risk for infection Rationale: Indwelling urinary catheters are a major source of infection (D). (A and B) are both problems that may require an indwelling catheter. (C) is not affected by an indwelling catheter.

When the health care provider diagnoses metastatic cancer and recommends a gastrostomy for an older female client in stable condition, the son tells the nurse that his mother must not be told the reason for the surgery because she "can't handle" the cancer diagnosis. Which legal principle is the court most likely to uphold regarding this client's right to informed consent? A. The family can provide the consent required in this situation because the older adult is in no condition to make such decisions. B. Because the client is mentally incompetent, the son has the right to waive informed consent for her. C. The court will allow the health care provider to make the decision to withhold informed consent under therapeutic privilege. D. If informed consent is withheld from a client, health care providers could be found guilty of negligence.

D. If informed consent is withheld from a client, health care providers could be found guilty of negligence. Rationale: Health care providers may be found guilty of negligence (D), specifically assault and battery, if they carry out a treatment without the client's consent. The client's condition is stable, so (A) is not a valid rationale. Advanced age does not automatically authorize the son to make all decisions for his mother, and there is no evidence that the client is mentally incompetent (B). Although (C) may have been upheld in the past, when paternalistic medical practice was common, today's courts are unlikely to accept it.

The nurse is preparing an older client for discharge. Which method is best for the nurse to use when evaluating the client's ability to perform a dressing change at home? A. Determine how the client feels about changing the dressing. B. Ask the client to describe the procedure in writing. C. Seek a family member's evaluation of the client's ability to change the dressing. D. Observe the client change the dressing unassisted

D. Observe the client change the dressing unassisted

A client becomes angry while waiting for a supervised break to smoke a cigarette outside and states, "I want to go outside now and smoke. It takes forever to get anything done here!" Which intervention is best for the nurse to implement? A. Encourage the client to use a nicotine patch. B. Reassure the client that it is almost time for another break. C. Have the client leave the unit with another staff. D. Review the schedule of outdoor breaks with the client.

D. Review the schedule of outdoor breaks with the client. Rationale: The best nursing action is to review the schedule of outdoor breaks (D) and provide concrete information about the schedule. (A) is contraindicated if the client wants to continue smoking. (B) is insufficient to encourage a trusting relationship with the client. (C) is preferential for this client only and is inconsistent with unit rules

The nurse learns that members of the nursing staff are uncomfortable with responding to client family members who are angry. In designing a teaching session to help the staff respond more effectively in these situations, which instructional strategy is best for the nurse to use? A. Return demonstration B. Journaling C. Analogies D. Role playing

D. Role playing

Which serum laboratory value should the nurse monitor carefully for a client who has a nasogastric (NG) tube to suction for the past week? A. White blood cell count B. Albumin C. Calcium D. Sodium

D. Sodium

The nurse finds a client crying behind a locked bathroom door. The client will not open the door. Which action should the nurse implement first? A. Instruct an unlicensed assistive personnel (UAP) to stay and keep talking to the client. B. Sit quietly in the client's room until the client leaves the bathroom. C. Allow the client to cry alone and leave the client in the bathroom. D. Talk to the client and attempt to find out why the client is crying.

D. Talk to the client and attempt to find out why the client is crying. Rationale: The nurse's first concern should be for the client's safety, so an immediate assessment of the client's situation is needed (D). (A) is incorrect; the nurse should implement the intervention. The nurse may offer to stay nearby after first assessing the situation more fully (B). Although (C) may be correct, the nurse should determine if the client's safety is compromised and offer assistance, even if it is refused.

A 20-year-old female client with a noticeable body odor has refused to shower for the last 3 days. She states, "I have been told that it is harmful to bathe during my period." Which action should the nurse take first? A. Accept and document the client's wish to refrain from bathing. B. Offer to give the client a bed bath, avoiding the perineal area. C. Obtain written brochures about menstruation to give to the client. D. Teach the importance of personal hygiene during menstruation with the client.

D. Teach the importance of personal hygiene during menstruation with the client. Rationale: Because a shower is most beneficial for the client in terms of hygiene, the client should receive teaching first (D), respecting any personal beliefs such as cultural or spiritual values. After client teaching, the client may still choose (A or B). Brochures reinforce the teaching (C).

A client has a nasogastric tube connected to low intermittent suction. When administering medications through the nasogastric tube, which action should the nurse do first? A. Clamp the nasogastric tube. B. Confirm placement of the tube. C. Use a syringe to instill the medications. D. Turn off the intermittent suction device.

D. Turn off the intermittent suction device. Rationale: The nurse should first turn off the suction (D) and then confirm placement of the tube in the stomach (B) before instilling the medications (C). To prevent immediate removal of the instilled medications and allow absorption, the tube should be clamped for a period of time (A) before reconnecting the suction.

The client in the outpatient clinic complains of experiencing hard, infrequent stools. Which instruction should the nurse provide this client? A: "Walk around the block, or further, every day." B: "Take an over-the-counter laxative every morning." C: "Try using over-the-counter enemas when needed." D: "Drink six to eight large glasses of water daily.

D: "Drink six to eight large glasses of water daily.

The nurse is planning care for a group of patients on a Med-Surg unit during night shift. Which patient should be closely monitored for sleep apnea? A: A woman with restless leg syndrome and COPD. B: A young woman taking Coumadin and has a diagnosis of insomnia. C: A male with a bleeding ulcer. D: A male with multiple problems including diabetes, HTN, and obesity.

D: A male with multiple problems including diabetes, HTN, and obesity.

The nurse is providing passive ROM exercises to the hip and knee for a client who is unconscious. After supporting the client's knee with one hand, what action should the nurse take next? A: Raise the bed to a comfortable working level. B: Bend the client's knee. C: Move the knee toward the chest as far as it will go. D: Cradle the client's heel.

D: Cradle the client's heel.

A 35 year old female client refuses to allow the nurse to insert an IV for a scheduled chemotherapy treatment. The patient states she is ready to go home and die. What intervention should the nurse initiate? A: Review the medical record for an advanced directive. B: Determine if the physician has written a DNR. C: Document that the client is being discharged against medical advice. D: Determine if the client is competent to refuse medical treatment.

D: Determine if the client is competent to refuse medical treatment.

A client on a prescribed full liquid diet has a nursing diagnosis of "Risk for impaired skin integrity related to reduced oral intake." What snack is best to provide this client? A: Beef broth or chicken broth. B: Purified lowfat milk. C: Apple or grapefruit juice. D: Ensure, a liquid supplement.

D: Ensure, a liquid supplement.

Which statement is an example of a correctly written nursing diagnosis? A: Altered tissue perfusion R/T congestive heart failure. B: Altered urinary elimination R/T urinary tract infection. C: Risk for impaired tissue integrity R/T client's refusal to turn. D: Ineffective coping R/T response to positive biopsy test results.

D: Ineffective coping R/T response to positive biopsy test results.

Which action should the nurse implement when using the confrontation technique during a vision exam? A: Use an Ophthalmoscope to watch the client's pupil constrict when a strong light is shown into it. B: Stand behind the client and direct the client to tell the nurse when an object enters the peripheral field of vision. C: Show the client a series of four cards with printing of various sizes and ask which card the client sees most clearly. D: Sit facing the client and while looking directly at the client's face, move an object inward from the periphery.

D: Sit facing the client and while looking directly at the client's face, move an object inward from the periphery.

The nurse assess an elderly, immobilized male patient, BP: 138/7, Temp: 96.9, urine output = 100ml concentrated urine in the last hour. He has increased respiratory secretions and wet lung sounds. Which nursing action is the most important to implement? A: Administer a PRN antihypertensive medication. B: Provide the patient with an extra blanket. C: Encourage increased fluid intake. D: Turn every 2 hours.

D: Turn every 2 hours.

The healthcare provider prescribes Haloperidol (Haldol) 1.5mg twice daily for a client with Tourette's syndrome. The drug is available in a solution labeled "2mg / ml" How many ml should the nurse administer?(enter numerical value only, If rounding is required round to the nearest Tenths).

0.75

The healthcare provider prescribes haloperidol (Haldol) 1.5mg twice daily for a client with Tourette's syndrome. The drug is available in a solution labeled, "2 mg/ml." How many ml should the nurse administer? (Round to the nearest hundredth.)

0.75

A client is receiving Ketorolac (Toradol) IM 45mg IM every 6 hours for post operative pain. The available 2ml vile is labeled, Toradol 30mg / ML. How many ML should the nurse administer? (enter numerical value only, If rounding is required round to the nearest Tenths).

1.5

A client is receiving ketorolac (Toradol) IM 45 mg IM 6 hours for postoperative pain. The available 2 ml vial is labeled , Toradol IM 30 mg/ml, How many should the nurse administer? (Round to the nearest tenth.)

1.5

The nurse is caring for a male client with diminished circulation in the lower extremities. The client washes his feet in the shower, but is unable to bend safely to dry his feet. While drying the client's feet, the nurse should emphasize the need to thoroughly dry which area of the feet? A) Between the toes. B) Around the ankles. C) On dorsal surfaces D) Over the heels.

A) Between the toes.

The nurse observes a newly admitted older adult female take short steps and walk very slowly while pushing a walker in front of her. What action should the nurse take in response to these observations? A) Complete a full fall risk assessment of the client. B) Teach the client to take longer steps at a faster pace. C) Suggest that the client use a wheelchair instead of a walker. D) Place the client on bed rest until the healthcare provider is notified.

A) Complete a full fall risk assessment of the client.

An older woman with end-stage heart disease is hospitalized for severe heart failure. She is alert, oriented, and requests that no heroic measures are implemented if her breathing stops. What action should the nurse take first? A) Discuss with the client her meaning of heroic measures. B) Obtain a "do not resuscitate" (DNR) prescription. C) Set up a family conference to discuss the clients. D) Consult the palliative care team about the client's care.

A) Discuss with the client her meaning of heroic measures.

Which landmarks are useful to the nurse when administering an intramuscular injection in ventrogluteal site? A) The greater trochanter and anterior superior iliac spine. B) The knee and greater trochanter. C) The upper, outer quadrant of the buttock. D) The deltoid muscle.

A) The greater trochanter and anterior superior iliac spine.

While reviewing the side effects of a newly prescribed medication, a 72-year- old client notes that one of the side effects is a reduction in sexual drive. Which is the best response by the nurse? A. "How will this affect your present sexual activity?" B. "How active is your current sex life?" C. "How has your sex life changed as you have become older?" D. "Tell me about your sexual needs as an older adult."

A. "How will this affect your present sexual activity?" Rationale:(A) offers an open-ended question most relevant to the client's statement. (B) does not offer the client the opportunity to express concerns. (C and D) are even less relevant to the client's statement.

A male client is laughing at a television program with his wife when the evening nurse enters the room. He says his foot is hurting and he would like a pain pill. How should the nurse respond? A. Ask him to rate his pain on a scale of 1 to 10. B. Encourage him to wait until bedtime so the pill can help him sleep. C. Attend to an acutely ill client's needs first because this client is laughing. D. Instruct him in the use of deep breathing exercises for pain control.

A. Ask him to rate his pain on a scale of 1 to 10. Rationale: Obtaining a subjective estimate of the pain experience by asking the client to rate his pain (A) helps the nurse determine which pain medication should be administered and also provides a baseline for evaluating the effectiveness of the medication. Medicating for pain should not be delayed so that it can be used as a sleep medication (B). (C) is judgmental. (D) should be used as an adjunct to pain medication, not instead of medication.

An older client who had abdominal surgery 3 days earlier was given a barbiturate for sleep and is now requesting to go to the bathroom. Which action should the nurse implement? A. Assist the client to walk to the bathroom and do not leave the client alone. B. Request that the UAP assist the client onto a bedpan. C. Ask if the client needs to have a bowel movement or void. D. Assess the client's bladder to determine if the client needs to urinate.

A. Assist the client to walk to the bathroom and do not leave the client alone Rationale: Barbiturates cause central nervous system (CNS) depression and individuals taking these medications are at greater risk for falls. The nurse should assist the client to the bathroom (A). A bedpan (B) is not necessary as long as safety is ensured. Whether the client needs to void or have a bowel movement, (C) is irrelevant in terms of meeting this client's safety needs. There is no indication that this client cannot voice her or his needs, so assessment of the bladder is not needed (D).

In assisting an older adult client prepare to take a tub bath, which nursing action is most important? A. Check the bath water temperature. B. Shut the bathroom door. C. Ensure that the client has voided. D. Provide extra towels.

A. Check the bath water temperature. Rationale: To prevent burns or excessive chilling, the nurse must check the bath water temperature (A). (B, C, and D) promote comfort and privacy and are important interventions but are of less priority than promoting safety.

In taking a client's history, the nurse asks about the stool characteristics. Which description should the nurse report to the health care provider as soon as possible? A. Daily black, sticky stool B. Daily dark brown stool C. Firm brown stool every other day D. Soft light brown stool twice a day

A. Daily black, sticky stool Rationale: Black sticky stool (melena) is a sign of gastrointestinal bleeding and should be reported to the health care provider promptly (A). (C) indicates constipation, which is a lesser priority. (B and D) are variations of normal.

A client has a nursing diagnosis of Altered sleep patterns related to nocturia. Which client instruction is important for the nurse to provide? A. Decrease intake of fluids after the evening meal. B. Drink a glass of cranberry juice every day. C. Drink a glass of warm decaffeinated beverage at bedtime. D. Consult the health care provider about a sleeping pill.

A. Decrease intake of fluids after the evening meal. Rationale: Nocturia is urination during the night. (A) is helpful to decrease the production of urine, thus decreasing the need to void at night. (B) helps prevent bladder infections. (C) may promote sleep, but the fluid will contribute to nocturia. (D) may result in urinary incontinence if the client is sedated and does not awaken to void.

A hospitalized client has had difficulty falling asleep for 2 nights and is becoming irritable and restless. Which action by the nurse is best? A. Determine the client's usual bedtime routine and include these rituals in the plan of care as safety allows. B. Instruct the UAP not to wake the client under any circumstances during the night. C. Place a "Do Not Disturb" sign on the door and change assessments from every 4 to every 8 hours. D. Encourage the client to avoid pain medication during the day, which might increase daytime napping.

A. Determine the client's usual bedtime routine and include these rituals in the plan of care as safety allows. Rationale: Including habitual rituals that do not interfere with the client's care or safety may allow the client to go to sleep faster and increase the quality of care (A). (B, C, and D) decrease the client's standard of care and compromise safety.

The nurse is using the Glasgow Coma Scale to perform a neurologic assessment. A comatose client winces and pulls away from a painful stimulus. Which action should the nurse take next? A. Document that the client responds to painful stimulus. B. Observe the client's response to verbal stimulation. C. Place the client on seizure precautions for 24 hours. D. Report decorticate posturing to the health care provider.

A. Document that the client responds to painful stimulus. Rationale: The client has demonstrated a purposeful response to pain, which should be documented as such (A). Response to painful stimulus is assessed after response to verbal stimulus, not before (B). There is no indication for placing the client on seizure precautions (C). Reporting (D) is nonpurposeful movement.

While conducting an intake assessment of an adult male at a community mental health clinic, the nurse notes that his affect is flat, he responds to questions with short answers, and he reports problems with sleeping. He reports that his life partner recently died from pneumonia. Which action is most important for the nurse to implement? A. Encourage the client to see the clinic's grief counselor. B. Determine if the client has a family history of suicide attempts. C. Inquire about whether the life partner was suffering from AIDS. D. Consult with the health care provider about the client's need for antidepressant medications.

A. Encourage the client to see the clinic's grief counselor. Rationale: The client is exhibiting normal grieving behaviors, so referral to a grief counselor (A) is the most important intervention for the nurse to implement. (B) is indicated, but is not a high-priority intervention. (C) is irrelevant at this time but might be important when determining the client's risk for contracting the illness. An antidepressant may be indicated (D), depending on further assessment, but grief counseling is a better action at this time because grief is an expected reaction to the loss of a loved one.

The nurse is aware that malnutrition is a common problem among clients served by a community health clinic for the homeless. Which laboratory value is the most reliable indicator of chronic protein malnutrition? A. Low serum albumin level B. Low serum transferrin level C. High hemoglobin level D. High cholesterol level

A. Low serum albumin level Rationale: Long-term protein deficiency is required to cause significantly lowered serum albumin levels (A). Albumin is made by the liver only when adequate amounts of amino acids (from protein breakdown) are available. Albumin has a long half-life, so acute protein loss does not significantly alter serum levels. (B) is a serum protein with a half-life of only 8 to 10 days, so it will drop with an acute protein deficiency. Neither (C or D) are clinical measures of protein malnutrition.

Based on the nursing diagnosis of Risk for infection, which intervention is best for the nurse to implement when providing care for an older incontinent client? A. Maintain standard precautions. B. Initiate contact isolation measures. C. Insert an indwelling urinary catheter. D. Instruct client in the use of adult diapers

A. Maintain standard precautions. Rationale: The best action to decrease the risk of infection in vulnerable clients is hand washing (A). (B) is not necessary unless the client has an infection. (C) increases the risk of infection. (D) does not reduce the risk of infection.

By rolling contaminated gloves inside-out, the nurse is affecting which step in the chain of infection? A. Mode of transmission B. Portal of entry C. Reservoir D. Portal of exit

A. Mode of transmission Rationale: The contaminated gloves serve as the mode of transmission (A) from the portal of exit (D) of the reservoir (C) to a portal of entry (B).

The nurse-manager of a skilled nursing (chronic care) unit is instructing UAPs on ways to prevent complications of immobility. Which intervention should be included in this instruction? A. Perform range-of-motion exercises to prevent contractures. B. Decrease the client's fluid intake to prevent diarrhea. C. Massage the client's legs to reduce embolism occurrence. D. Turn the client from side to back every shift.

A. Perform range-of-motion exercises to prevent contractures. Rationale: Performing range-of-motion exercises (A) is beneficial in reducing contractures around joints. (B, C, and D) are all potentially harmful practices that place the immobile client at risk of complications.

The nurse prepares to insert a nasogastric tube in a client with hyperemesis who is awake and alert. Which intervention(s) is(are) correct? (Select all that apply.) A. Place the client in a high Fowler's position. B. Help the patient assume a left side-lying position. C. Measure the tube from the tip of the nose to the umbilicus. D. Instruct the client to swallow after the tube has passed the pharynx. E. Assist the client in extending the neck back so the tube may enter the larynx.

A. Place the client in a high Fowler's position. D. Instruct the client to swallow after the tube has passed the pharynx. Rationale: (A and D) are the correct steps to follow during nasogastric intubation. Only the unconscious or obtunded client should be placed in a left side-lying position (B). The tube should be measured from the tip of the nose to behind the ear and then from behind the ear to the xiphoid process (C). The neck should only be extended back prior to the tube passing the pharynx and then the client should be instructed to position the neck forward (E).

Which step(s) should the nurse take when administering ear drops to an adult client? (Select all that apply.) A. Place the client in a side-lying position. B. Pull the auricle upward and outward. C. Hold the dropper 6 cm above the ear canal. D. Place a cotton ball into the inner canal. E. Pull the auricle down and back.

A. Place the client in a side-lying position. B. Pull the auricle upward and outward. Rationale: The correct answers (A and B) are the appropriate administration of ear drops. The dropper should be held 1 cm (½ inch) above the ear canal (C). A cotton ball should be placed in the outermost canal (D). The auricle is pulled down and back for a child younger than 3 years of age, but not an adult (E).

When assessing a client who starts to wheeze which related data should the nurse obtain? A. Precipitating factors B. Body Temperature C. Presence of radiation D. Heart sounds

A. Precipitating factors

A nurse is working in an occupational health clinic when an employee walks in and states that he was struck by lightning while working in a truck bed. The client is alert but reports feeling faint. Which assessment will the nurse perform first? A. Pulse characteristics B. Open airway C. Entrance and exit wounds D. Cervical spine injury

A. Pulse characteristics Rationale: Lightning is a jolt of electrical current and can produce a "natural" defibrillation, so assessment of the pulse rate and regularity (A) is a priority. Because the client is talking, he has an open airway (B), so that assessment is not necessary. Assessing for (C and D) should occur after assessing for adequate circulation.

When emptying 350 mL of pale yellow urine from a client's urinal, the nurse notes that this is the first time the client has voided in 4 hours. Which action should the nurse take next? A. Record the amount on the client's fluid output record. B. Encourage the client to increase oral fluid intake. C. Notify the health care provider of the findings. D. Palpate the client's bladder for distention.

A. Record the amount on the client's fluid output record. Rationale: The amount and appearance of the client's urine output is within normal limits, so the nurse should record the output (A), but no additional action is needed (B, C, and D).

Which client is most likely to be at risk for spiritual distress? A. Roman Catholic woman considering an abortion B. Jewish man considering hospice care for his wife C. Seventh-Day Adventist who needs a blood transfusion D. Muslim man who needs a total knee replacement

A. Roman Catholic woman considering an abortion Rationale: In the Roman Catholic religion, any type of abortion is prohibited (A), so facing this decision may place the client at risk for spiritual distress. There is no prohibition of hospice care for members of the Jewish faith (B). Jehovah's Witnesses prohibit blood transfusions, not Seventh-Day Adventists (C). There is no conflict in the Muslim faith with regard to joint replacement (D).

Which nonverbal action should the nurse implement to demonstrate active listening? A. Sit facing the client. B. Cross arms and legs. C. Avoid eye contact. D. Lean back in the chair.

A. Sit facing the client. Rationale: Active listening is conveyed using attentive verbal and nonverbal communication techniques. To facilitate therapeutic communication and attentiveness, the nurse should sit facing the client (A), which lets the client know that the nurse is there to listen. Active listening skills include postures that are open to the client, such as keeping the arms open and relaxed, not (B), and leaning toward the client, not (D). To communicate involvement and willingness to listen to the client, eye contact should be established and maintained (C).

The charge nurse observes a new graduate's performance of wound care. Which technique indicates that the employee is effectively cleansing the wound? A. Starts at the wound site and moves outward using circular motions. B. Cleanses from the outer area of the wound toward the center C. Uses a sterile swab to go over the wound site twice. D. Scrubs wound vigorously for at least two minutes

A. Starts at the wound site and moves outward using circular motions.

The nurse is obtaining a lie-sit-stand blood pressure reading on a client. Which action is most important for the nurse to implement? A. Stay with the client while the client is standing. B. Record the findings on the graphic sheet in the chart. C. Keep the blood pressure cuff on the same arm. D. Record changes in the client's pulse rate.

A. Stay with the client while the client is standing. Rationale: Although all these measures are important, (A) is most important because it helps ensure client safety. (B) is necessary but does not have the priority of (A). (C and D) are important measures to ensure accuracy of the recording but are of less importance than providing client safety.

A client at an outpatient clinic submits a clean- catch midstream urine specimen for a routine urinalysis. In later review of the client's medical record, which data indicates to the nurse that the specimen collection should be repeated? A. The urine specimen shows multiple organisms in low colony counts. B. The client reported eating a meal before voiding the urine specimen C. There was a total of 30 ml of urine voided into the specimen cup D. The medical record indicates the client is allergic to most antibiotics

A. The urine specimen shows multiple organisms in low colony counts.

The nurse enters the room of a client with Clostridium difficile infection to administer an IV antibiotic. The UAP is in the room cleaning the client's buttocks and states the client has been incontinent with diarrhea. The UAP is wearing gloves but not a gown. What action should the nurse implement first? A: Advise the UAP to put on a gown. B: Observe the appearance of the diarrhea. C: Hang the scheduled dose of antibiotic. D: Assess the client's skin integrity.

A: Advise the UAP to put on a gown.

A 24-hour urine collection is in progress. The client tells the nurse that the last voiding was accidentally flushed instead of saved in the container. What interventions should the nurse initiate? A: Discard the urine and start another 24 hour period. B: Notify the charge nurse of the problem. C: Notify the healthcare provider of the situation. D: Add another hour to the urine collection period.

A: Discard the urine and start another 24 hour period.

A client is 2 days post-op from a thoracic surgery and is complaining of incisional pain. The client last received pain medication 2 hours ago. He is rating his pain a 5 on a 1-10 scale. After calling the provider, what is the nurse's next action? A: Encourage the client to use guided imagery and slow, rhythmic breathing. B: Provide 20 minutes of back massage. C: Encourage the client to watch TV. D: Place a hot water circulation device, such as an Aqua K pad.

A: Encourage the client to use guided imagery and slow, rhythmic breathing.

A high school senior is complaining of a persistent cough and admits to smoking 10 to 15 cigarettes daily for the past year. He is convinced that he is hopelessly addicted to tobacco since he tried unsuccessfully to quit smoking last week. Which intervention is best for the nurse to implement? A: Encourage the student to associate with non-smokers only while attempting to stop smoking. B: Tell the student that he is still young and should continue to try various smoking cessation methods. C: Describe cigarette smoking as a habit that requires a strong will to overcome its addictiveness. D: Provide the student with the latest research data describing the long-term effects of tobacco use.

A: Encourage the student to associate with non-smokers only while attempting to stop smoking. Rationale: A. It is difficult to cease smoking when surrounded by those who smoke, and adolescents are particularly influenced by peers, so (A) is the most important intervention for the nurse to implement. (B) is not likely to be helpful and offers no concrete suggestions for smoking cessation. (C) is condescending. Risks associated with smoking must already be known to this adolescent who is already attempting to stop the habit (D). Category: Fundamentals

What clinical manifestations would indicate to the nurse that a patient is possibly experiencing orthostatic hypotension? A: Nausea B: Lightheadedness C: Dizziness D: Patient c/o seeing spots. E: Flushing face. F: Bradycardia.

A: Nausea B: Lightheadedness C: Dizziness D: Patient c/o seeing spots.

After reviewing the admission assessment of a client with chronic pain, which interventions should the nurse include in this client's plan of care?(SATA) A: Provide comfort measures such as topical warm application and tactile massage. B: Encourage increased fluid intake and measure urinary output Q8 hours. C: Implement a 24-hour schedule of routine administration of prescribed analgesic. D: Determine client's subjective measure of pain using a numerical pain scale. E: Assist the client to ambulate as much as possible during waking hours.

A: Provide comfort measures such as topical warm application and tactile massage. C: Implement a 24-hour schedule of routine administration of prescribed analgesic. D: Determine client's subjective measure of pain using a numerical pain scale. E: Assist the client to ambulate as much as possible during waking hours.

An elderly woman comes to the clinic because of vaginal bleeding. The healthcare provider finds a vaginal tear, which the client reports is likely to have occurred during unprotected sexual intercourse. Which content is most important for the nurse to include in the client's teaching plan? A: The importance of using vaginal lubricants. B: Methods used to practice safe sex. C: Information about alternative ways to express sexuality. D: Intercourse positions that help prevent tears.

A: The importance of using vaginal lubricants.

It is most important for a nurse to recalculate a patient's Braden score who develops which problem? A: Urinary incontinence. B: Hypoactive bowel sounds. C: Weakened cough reflex. D: 2+ pitting edema to both legs.

A: Urinary incontinence.

The healthcare provider prescribes bladder irrigation to maintain patency of a client's indwelling urinary catheter. Which intervention should the nurse implement? A: Use a sterile syringe to irrigate with NS 20 ml. B: Use an infusion pump to slowly irrigate the indwelling catheter. C: Clamp the catheter for 30 minutes prior to irrigating with NS. D: Power flush with NS 60 ml to remove mucous.

A: Use a sterile syringe to irrigate with NS 20 ml.

What information is most important for the nurse to obtain in determining a client's need for referral for obesity counseling? A) Body weight 10% over ideal body weight. B) Body mass index greater than 35. C) Daily caloric intake of 3500 calories. D) Client's expressed desire to lose 50 pounds.

B) Body mass index greater than 35.

The nurse observes that there are reddened areas on the cheekbones of a client receiving oxygen per nasal cannula at 3L/minute, and the client's oxygen saturation level is 92%. What intervention should the nurse implement? A) Decrease the flow rate to 1 L/minute. B) Discontinue the use of the nasal cannula. C) Apply lubricant to the cannula tubing. D) Place padding around the cannula tubing.

B) Discontinue the use of the nasal cannula.

A male client has right-sided hemiplegia following a left cerebrovascular accident (CVA). His sitting balance has improved, and he is now able to sit in a wheelchair. To assist the client in transferring from the bed to a wheelchair, what action should the nurse take? A) Have the client put both arms around the nurse's neck for support. B) Place the wheelchair on the client's left side. C) Instruct the client to look at his feet. D) Instruct the client to take slow, deep breaths while transferring

B) Place the wheelchair on the client's left side.

The nurse is caring for a hospitalized client who was placed in restraints due to confusion. The family removes the restraints while they are with the client. When the family leaves, what action should the nurse take first? A) Apply the restraints to maintain the client's safety. B) Reassess the client to determine the need for continuing restraints. C) Document the time the family left and continue to monitor the client. D) Call the healthcare provider for a new prescription.

B) Reassess the client to determine the need for continuing restraints.

A client has begun a long-term maintenance therapy with lithium, which has a narrow therapeutic index. Which adverse effect is most important for nurse to include in the teaching plan? A) Dependence. B) Toxicity. C) Interaction. D) Tolerance.

B) Toxicity.

The nurse is instructing a client in the proper use of a metered-dose inhaler. Which instruction should the nurse provide the client to ensure the optimal benefits from the drug? A. "Fill your lungs with air through your mouth and then compress the inhaler." B. "Compress the inhaler while slowly breathing in through your mouth." C. "Compress the inhaler while inhaling quickly through your nose." D. "Exhale completely after compressing the inhaler and then inhale."

B. "Compress the inhaler while slowly breathing in through your mouth." Rationale: The medication should be inhaled through the mouth simultaneously with compression of the inhaler (B). This will facilitate the desired destination of the aerosol medication deep in the lungs for an optimal bronchodilation effect. (A, C, and D) do not allow for deep lung penetration.

The nurse is counting a client's respiratory rate. During a 30-second interval, the nurse counts six respirations and the client coughs three times. In repeating the count for a second 30-second interval, the nurse counts eight respirations. Which respiratory rate should the nurse document? A. 14 B. 16 C. 17 D. 28

B. 16

Ten minutes after signing an operative permit for a fractured hip, an older client states, "The aliens will be coming to get me soon!" and falls asleep. Which action should the nurse implement next? A. Make the client comfortable and allow the client to sleep. B. Assess the client's neurologic status. C. Notify the surgeon about the comment. D. Ask the client's family to co-sign the operative permit.

B. Assess the client's neurologic status. Rationale: This statement may indicate that the client is confused. Informed consent must be provided by a mentally competent individual, so the nurse should further assess the client's neurologic status (B) to be sure that the client understands and can legally provide consent for surgery. (A) does not provide sufficient follow-up. If the nurse determines that the client is confused, the surgeon must be notified (C) and permission obtained from the next of kin (D).

The nurse is instructing a client with cholecystitis regarding diet choices. Which meal best meets the dietary needs of this client? A. Steak, baked beans, and a salad B. Broiled fish, green beans, and an apple C. Pork chops, macaroni and cheese, and grapes D. Avocado salad, milk, and angel food cake

B. Broiled fish, green beans, and an apple Rationale: Clients with cholecystitis (inflammation of the gallbladder) should follow a low-fat diet, such as (B). (A) is a high-protein diet and (C and D) contain high-fat foods, which are contraindicated for this client.

A 65-year-old client who attends an adult daycare program and is wheelchair-mobile has redness in the sacral area. Which instruction is most important for the nurse to provide? A. Take a vitamin supplement tablet once a day. B. Change positions in the chair at least every hour. C. Increase daily intake of water or other oral fluids. D. Purchase a newer model wheelchair.

B. Change positions in the chair at least every hour. Rationale The most important teaching is to change positions frequently (B) because pressure is the most significant factor related to the development of pressure ulcers. Increased vitamin and fluid intake (A and C) may also be beneficial promote healing and reduce further risk. (D) is an intervention of last resort because this will be very expensive for the client.

The nurse assesses a 2-year-old who is admitted for dehydration and finds that the peripheral IV rate by gravity has slowed, even though the venous access site is healthy. What should the nurse do next? A. Apply a warm compress proximal to the site. B. Check for kinks in the tubing and raise the IV pole. C. Adjust the tape that stabilizes the needle. D. Flush with normal saline and recount the drop rate.

B. Check for kinks in the tubing and raise the IV pole. Rationale: The nurse should first check the tubing and height of the bag on the IV pole (B), which are common factors that may slow the rate. Gravity infusion rates are influenced by the height of the bag, tubing clamp closure or kinks, needle size or position, fluid viscosity, client blood pressure (crying in the pediatric client), and infiltration. Venospasm can slow the rate and often responds to warmth over the vessel (A), but the nurse should first adjust the IV pole height. The nurse may need to adjust the stabilizing tape on a positional needle (C) or flush the venous access with normal saline (D), but less invasive actions should be implemented first.

The home health nurse is reviewing the personal care needs of an elderly client who lives alone. Which client assessment findings indicate the need to assign an unlicensed assistive personal (UAP) to provide routine foot care and file the client's toenails? (Select all that apply). A. Shuffling gait. B.Diminished visual acuity. C. Syncope when bending. D. hands tremors. E.Urinary incontinence

B. Diminished visual acuity. C. Syncope when bending. D. hands tremors.

The nurse observes the skin over a client's greater trochanter as seen in the picture. What actions should the nurse implement? (select all that apply) A. Remove the eschar before applying and securing a hydrocolliod B. Prepare to implement a pressure redistribution mattress C. Obtain a specimen of the site for culture and sensitivity D. Instruct the Unlicensed assistive personnel to frequently offer oral fluids E. Explain to the client that the wound needs debridement

B. Prepare to implement a pressure redistribution mattress D. Instruct the Unlicensed assistive personnel to frequently offer oral fluids

When turning an immobile bedridden client without assistance, which action by the nurse best ensures client safety? A. Securely grasp the client's arm and leg. B. Put bed rails up on the side of bed opposite from the nurse. C. Correctly position and use a turn sheet. D. Lower the head of the client's bed slowly.

B. Put bed rails up on the side of bed opposite from the nurse. Rationale Because the nurse can only stand on one side of the bed, bed rails should be up on the opposite side to ensure that the client does not fall out of bed (B). (A) can cause client injury to the skin or joint. (C and D) are useful techniques while turning a client but have less priority in terms of safety than use of the bed rails.

During evacuation of a group of clients from a medical unit because of a fire, the nurse observes an ambulatory client walking alone toward the stairway at the end of the hall. Which action should the nurse take? A. Assign an unlicensed assistive personnel to transport the client via a wheelchair. B. Remind the client to walk carefully down the stairs until reaching a lower floor. C. Ask the client to help by assisting a wheelchair-bound client to a nearby elevator. D. Open the closest fire doors so that ambulatory clients can evacuate more rapidly.

B. Remind the client to walk carefully down the stairs until reaching a lower floor. Rationale: During evacuation of a unit because of fire, ambulatory clients should be evacuated via the stairway if at all possible and reminded to walk carefully (B). Ambulatory clients do not require the assistance of a wheelchair to be evacuated (A). Elevators (C) should not be used during a fire and fire doors should be kept closed (D) to help contain the fire.

The nurse selects the best site for insertion of an IV catheter in the client's right arm. Which documentation should the nurse use to identify placement of the IV access? A. Left brachial vein B. Right cephalic vein C. Dorsal side of the right wrist D. Right upper extremity

B. Right cephalic vein Rationale: The cephalic vein is large and superficial and identifies the anatomic name of the vein that is accessed, which should be included in the documentation (B). The basilic vein of the arm is used for IV access, not the brachial vein (A), which is too deep to be accessed for IV infusion. Although veins on the dorsal side of the right wrist (C) are visible, they are fragile and using them would be painful, so they are not recommended for IV access. (D) is not specific enough for documenting the location of the IV access.

After a needlestick occurs while removing the cap from a sterile needle, which action should the nurse implement? A. Complete an incident report. B. Select another sterile needle. C. Disinfect the needle with an alcohol swab. D. Notify the supervisor of the department immediately.

B. Select another sterile needle. Rationale: After a needlestick, the needle is considered used, so the nurse should discard it and select another needle (B). Because the needle was sterile when the nurse was stuck and the needle was not in contact with any other person's body fluids, the nurse does not need to complete an incident report (A) or notify the occupational health nurse (D). Disinfecting a needle with an alcohol swab (C) is not in accordance with standards for safe practice and infection control.

Which action should the nurse implement when providing wound care instructions to a client who does not speak English? A. Ask an interpreter to provide wound care instructions. B. Speak directly to the client, with an interpreter translating. C. Request the accompanying family member to translate. D. Instruct a bilingual employee to read the instructions.

B. Speak directly to the client, with an interpreter translating. Rationale: Wound care instructions should be given directly to the client by the nurse with an interpreter (B) who is trained to provide accurate and objective translation in the client's primary language, so that the client has the opportunity to ask questions during the teaching process. The interpreter usually does not have any health care experience, so the nurse must provide client teaching (A). Family members should not be used to translate instructions (C) because the client or family member may alter the instructions during conversation or be uncomfortable with the topics discussed. The employee should be a trained interpreter (D) to ensure that the nurse's instructions are understood accurately by the client.

The nurse teaches the use of a gait belt to a male caregiver whose wife has right-sided weakness and needs assistance with ambulation. The caregiver performs a return demonstration of the skill. Which observation indicates that the caregiver has learned how to perform this procedure correctly? A. Standing on his wife's strong side, the caregiver is ready to hold the gait belt if any evidence of weakness is observed. B. Standing on his wife's weak side, the caregiver provides security by holding the gait belt from the back. C. Standing behind his wife, the caregiver provides balance by holding both sides of the gait belt. D. Standing slightly in front and to the right of his wife, the caregiver guides her forward by gently pulling on the gait belt.

B. Standing on his wife's weak side, the caregiver provides security by holding the gait belt from the back. Rationale: His wife is most likely to lean toward the weak side and needs extra support on that side and from the back (B) to prevent falling. (A, C, and D) provide less security for her.

A female nurse is assigned to care for a close friend, who says, "I am worried that friends will find out about my diagnosis." The nurse tells her friend that legally she must protect a client's confidentiality. Which resource describes the nurse's legal responsibilities? A. Code of Ethics for Nurses B. State Nurse Practice Act C. Patient's Bill of Rights D. ANA Standards of Practice

B. State Nurse Practice Act Rationale The State Nurse Practice Act (B) contains legal requirements for the protection of client confidentiality and the consequences for breaches in confidentiality. (A) outlines ethical standards for nursing care but does not include legal guidelines. (C and D) describe expectations for nursing practice but do not address legal implications.

The nurse is assessing several clients prior to surgery. Which factor in a client's history poses the greatest threat for complications to occur during surgery? A. Taking birth control pills for the past 2 years B. Taking anticoagulants for the past year C. Recently completing antibiotic therapy D. Having taken laxatives PRN for the last 6 months

B. Taking anticoagulants for the past yea Rationale: Anticoagulants (B) increase the risk for bleeding during surgery, which can pose a threat for the development of surgical complications. The health care provider should be informed that the client is taking these drugs. Although clients who take birth control pills (A) may be more susceptible to the development of thrombi, such problems usually occur postoperatively. A client with (C or D) is at less of a surgical risk than (B).

A seriously ill female client tells the nurse, "I am so tired and in so much pain! Please help me to die." Which is the best response for the nurse to provide? A. Administer the prescribed maximum dose of pain medication. B. Talk with the client about her feelings related to her own death. C. Collaborate with the health care provider about initiating antidepressant therapy. D. Refer the client to the ethics committee of her local health care facility.

B. Talk with the client about her feelings related to her own death. Rationale: The nurse should first assess the client's feelings about her death and determine the extent to which this statement expresses her true feelings (B). The client may need additional pain management, but further assessment is needed before implementing (A). (C and D) are both premature interventions and should not be implemented until further assessment is obtained.

The nurse observes a UAP taking a client's blood pressure in the lower extremity. Which observation of this procedure requires the nurse's intervention? A. The cuff wraps around the girth of the leg. B. The UAP auscultates the popliteal pulse with the cuff on the lower leg. C. The client is placed in a prone position. D. The systolic reading is 20 mm Hg higher than the blood pressure in the client's arm.

B. The UAP auscultates the popliteal pulse with the cuff on the lower leg. Rationale: When obtaining the blood pressure in the lower extremities, the popliteal pulse is the site for auscultation when the blood pressure cuff is applied around the thigh. The nurse should intervene with the UAP who has applied the cuff on the lower leg (B). (A) ensures an accurate assessment, and (C) provides the best access to the artery. Systolic pressure in the popliteal artery is usually 10 to 40 mm Hg higher (D) than in the brachial artery.

The health care provider has changed a client's prescription from the PO to the IV route of administration. The nurse should anticipate which change in the pharmacokinetic properties of the medication? A. The client will experience increased tolerance to the drug's effects and may need a higher dose. B. The onset of action of the drug will occur more rapidly, resulting in a more rapid effect. C. The medication will be more highly protein-bound, increasing the duration of action. D. The therapeutic index will be increased, placing the client at greater risk for toxicity.

B. The onset of action of the drug will occur more rapidly, resulting in a more rapid effect. Rationale: Because the absorptive process is eliminated when medications are administered via the IV route, the onset of action is more rapid, resulting in a more immediate effect (B). Drug tolerance (A), protein binding (C), and the drug's therapeutic index (D) are not affected by the change in route from PO to IV. In addition, an increased therapeutic index reduces the risk of drug toxicity.

The nurse identifies a potential for infection in a patient with partial- thickness (second-degree) and full-thickness (third-degree) burns. What intervention has the highest priority in decreasing the client's risk of infection? A. Administration of plasma expanders B. Use of careful hand washing technique C. Application of a topical antibacterial cream D. Limiting visitors to the client with burns

B. Use of careful hand washing technique Rationale: Careful hand washing technique (B) is the single most effective intervention for the prevention of contamination to all clients. (A) reverses the hypovolemia that initially accompanies burn trauma but is not related to decreasing the proliferation of infective organisms. (C and D) are recommended by various burn centers as possible ways to reduce the chance of infection. (B) is a proven technique to prevent infection.

When assisting a client from the bed to a chair, which procedure is best for the nurse to follow? A. Place the chair parallel to the bed, with its back toward the head of the bed and assist the client in moving to the chair. B. With the nurse's feet spread apart and knees aligned with the client's knees, stand and pivot the client into the chair. C. Assist the client to a standing position by gently lifting upward, underneath the axillae. D. Stand beside the client, place the client's arms around the nurse's neck, and gently move the client to the chair.

B. With the nurse's feet spread apart and knees aligned with the client's knees, stand and pivot the client into the chair. Rationale:(B) describes the correct positioning of the nurse and affords the nurse a wide base of support while stabilizing the client's knees when assisting to a standing position. The chair should be placed at a 45-degree angle to the bed, with the back of the chair toward the head of the bed (A). Clients should never be lifted under the axillae (C); this could damage nerves and strain the nurse's back. The client should be instructed to use the arms of the chair and should never place his or her arms around the nurse's neck (D); this places undue stress on the nurse's neck and back and increases the risk for a fall.

Which assessment finding is most significant in determining the level of assistance a client requires with personal care? A: 2+ pitting edema in the lower extremities. B: Disorientation to person, place, and time. C: A red rash in the groin area. D: Firm abdomen with hypoactive bowel sounds.

B: Disorientation to person, place, and time.

The nurse is preparing to irrigate the client's indwelling urinary catheter using an open technique. What action should the nurse take after applying gloves? A: Empty the client's urinary drainage bag. B: Draw up the irrigating solution into the syringe. C: Secure the client's catheter to the drainage tubing. D: Use aseptic technique to instill the irrigating solution.

B: Draw up the irrigating solution into the syringe.

The grandmother of a young adult male admitted to the psychiatric unit yesterday requests information about her grandson's treatment plan. Before answering the family member's questions, what action should the nurse take? A: Ask the client if he wants this information shared with his grandmother. B: Ensure that the signed release of information includes the grandmother. C: Consult with the healthcare provider before sharing the information. D: Reassure the grandmother by providing an honest response.

B: Ensure that the signed release of information includes the grandmother.

When caring for an immobile client, what nursing diagnosis has the highest priority? A: Risk for fluid volume deficit. B: Impaired gas exchange. C: Risk for impaired skin integrity. D: Altered tissue perfusion.

B: Impaired gas exchange.

The nurse explains that which of the following are goals of palliative care? A: Delays death. B: Offers a support system. C: Provides pain management. D: Focuses only on the patient, not the family. E: Manages symptoms of disease. F: Enhances quality of life.

B: Offers a support system C: Provides pain management. E: Manages symptoms of disease. F: Enhances quality of life.

What action should the nurse implement to prevent the formation of a sacral ulcer for a client who is immobile? A: Maintain in a lateral position using protective wrist and vest devices. B: Position prone with a small pillow below the diaphragm. C: Raise the head and knee gatch when lying in a supine position. D: Transfer into a wheelchair close to the nurses's station for observation.

B: Position prone with a small pillow below the diaphragm.

A confused elderly male client is having trouble sleeping at night and is sometimes found wandering in the hallway. What nursing intervention should the nurse implement first? A: Apply wrist restraints to prevent wandering. B: Provide a back rub at bedtime. C: Leave the door to his room open slightly. D: Administer a PRN sedative prescription.

B: Provide a back rub at bedtime.

Two nurses assess a client for a pulse deficit and count an apical pulse of 72 beats/minute and a radial pulse of 88 beats/minute. What action should the nurse take first? A: Obtain a second pulse deficit reading. B: Report the results to the healthcare provider. C: Measure the client's blood pressure. D: Document a pulse deficit of 16 bpm.

B: Report the results to the healthcare provider.

The nurse is administering an intermittent infusion of an antibiotic to a client whose IV access is an antecubital saline lock. After the nurse opens the roller clamp on the IV tubing, the alarm on the infusion pump indicates an obstruction. What action should the nurse take first? A: Check for a blood return. B: Reposition the client's arm. C: Remove the IV site dressing. D: Flush the lock with saline.

B: Reposition the client's arm.

The female client who is one day post mastectomy is crying when the nurse enters the room. What action should the nurse take? A: Remain quietly by the door until the client stops crying. B: Stay with the client in silence while touching her forearm. C: Ask the client if she would like her clergy notified. D: Tell the client it is normal to cry after surgery.

B: Stay with the client in silence while touching her forearm.

A male client with chronic debilitating heart disease asks the nurse to help him die because he believes that he will be better off dead rather than living under the current circumstances. The nurse supports the client and considers providing the family with a dose of medications that can result in the client's death. If the nurse acts on this intention, what is the most likely consequence? A: The nurse's actions will be rewarded by supporters of the right-to-die. B: The nurse will be prosecuted for the murder of the client. C: The client's family will defend the nurse who acted with empathy. D: The employing agency will be held legally responsible for the death.

B: The nurse will be prosecuted for the murder of the client.

Prior to attempting a digital removal of a fecal impaction, it is most important to perform which assessment? A: Bowel sounds. B: Vital Signs. C: Breath sounds. D: Abdominal girth.

B: Vital Signs.

To assess the quality of an adult client's pain, what approach should the nurse use? A) Observe body language and movement. B) Provide a numeric pain scale. C) Ask the client to describe the pain. D) Identify effective pain relief measures.

C) Ask the client to describe the pain

A client with limited tolerance for activity needs to walk in the hallway with assistance. Which instructions should the nurse give to the unlicensed assistive personnel (UAP) who assisting with client's care? (Select all that apply.) A) Instruct the client about signs of orthostatic hypertension B) Determine if the client needs to have a gait belt applied C) Measure the client's vital signs before the client walks. D) Offer to assist the client to void prior to walking in the hall. E) Report the onset of any dizziness or light headedness.

C) Measure the clients' vital signs before the client walks. D) Offer to assist the client to void prior to walking in the hall. E) Report the onset of any dizziness or light headedness.

The home care nurse is teaching a client how to change the dressing on a new venous stasis ulcer. The client has a history of a deep vein thrombosis and is allergic to latex. When removing the adhesive bandages, the nurse observes skin redness surrounding the draining wound. What action should the nurse implement? A) Replace dressing with cotton pads and silk tape. B) Measure and compare ankle-brachial pressure index. C) Obtain sample of the drainage for culture. D) Apply an antibiotic ointment to the wound.

C) Obtain sample of the drainage for culture.

A client is in contact isolation due to stage IV coccyx wound infected with methicillin resistant staphylococcus aureus (MRSA). The nurse plans interventions to prevent multiple re-entries to the client's room. In which order should the nurse perform the interventions? A) Change coccyx dressing, perform tracheostomy care, restart the IV. B) Perform tracheostomy care, change coccyx dressing, restart the IV. C) Restart the IV, perform tracheotomy care, change coccyx dressing. D) Change coccyx dressing, restart the IV, perform tracheostomy care.

C) Restart the IV, perform tracheotomy care, change coccyx dressing.

A client is admitted with Pneumonia and has a recent history of Methicilline-resistance Staphylococcus aureus (MRSA). The Client is placed in isolation. While caring for the client, which iems should the nurse place in a designated bio-hazard bag before it is removed from the room? a. A sputum specimen. b. Paper mask and gown. c. The nurse's stethoscope. d. Bed linens.

a. A sputum specimen. (BODILY FLUIDS = BIOHAZARD)

A postoperative client has three different PRN analgesics prescribed for different levels of pain. The nurse inadvertently administers a dose that is not within the prescribed parameters. What actions should the nurse take first? a. Access for side effects of the medication. b. Document the client's responses. c. complete a medication error report. d. Determine if the pain was relieved.

a. Access for side effects of the medication.

The community healthcare nurse is making a home visit when the client, who is sitting at the kitchen table, begins to have a seizure. What action should the nurse take first? a. Assist the client to the floor. b. Access the client's vital signs. c. Call 911 for an ambulance. d. Remove nearby furniture.

a. Assist the client to the floor.

While suctioning a client's nasopharynx the nurse observes that the client's oxygen saturation remains at 94% which is the same reading obtained prior to starting the procedure. What action should the nurse take in response to this finding? a. Complete the intermittent suction of the nasopharynx. b. Reposition the pulse oximeter clip to obtain a new reading. c. Stop suctioning until the pulse oximeter reading is above 95%. d. Apply an oxygen mask over the client's nose and mouth

a. Complete the intermittent suction of the nasopharynx.

The nurse measures the client's blood pressure (BP) and notes that it is significantly higher than the previous reading. What should the nurse do next? (Select all that apply). a. Determine the client's activities and feelings prior to the BP measurement. b. Retake the Client's blood pressure in the opposite arm c. Assign the unlicensed assistive personnel to recheck the BP in an hour. (not the answer because it should be rechecked sooner) d. Ask another nurse to assist in assessing for an apical-radial pulse deficit. e. Immediately take two more readings on the same arm.

a. Determine the client's activities and feelings prior to the BP measurement. b. Retake the Client's blood pressure in the opposite arm

A male client who had emergency gallbladder surgery yesterday is getting ready for discharge. The nurse knows that the client speaks very little English. When teaching wound care, which method should the nurse use to evaluate the client's understanding of self-care at home? a. Have the client demonstrate prescribed wound care. b. Provide written instructions in the client's native language. c. Have an interpreter repeat the wound care instructions. d. After each instruction, ask the client if he understands.

a. Have the client demonstrate prescribed wound care.

The nurse is preparing a teaching plan for a client with low back pain. Which sleeping position should be included in the teaching? a. Side-lying with hips and knees flexed. b. Supine with hips and knees and neutral straight position. c. Head of bed elevated to 30 degrees. d. Prone with a pillow under the lower abdomen.

a. Side-lying with hips and knees flexed.

The unlicensed assistive personnel (UAP) describes the appearance of the bowel movement of several clients. Which description warrants additional follow up by the nurse? (select all that applies). a. Solid with red streaks. b. Brown liquid. c. Multiple hard pellets. d. Formed but soft. e. Tarry appearance.

a. Solid with red streaks. c. Multiple hard pellets. e. Tarry appearance.

A client who lives in an assisted living facility; develops cognitive impairment following a stroke. Informed consent is needed to provide additional nursing services. Who should the nurse contact? a. The client's youngest son was identified by family members as the family's spokesperson. b. A daughter-in-law designated as the client's durable power of attorney (DPOA) c. The client's spouse who lives in the independent living unit of the facility. d. The client's oldest living child, a lawyer who is visiting from out of town.

b. A daughter-in-law designated as the client's durable power of attorney (DPOA)

The nurse prefers to implement a prescription for oxygen at 4 L/minute per nasal cannula. For a client with an oxygen saturation of 90%. The nurse observes the flow meter set up provided by the respiratory therapist, as seen in the picture. What action should the nurse take next? a. Adjust the flow rate to 4 L/minute b. Attach oxygen tubing to the flow meter. c. Drain the water out of the humidifier. d. Document the presence of breath sounds.

b. Attach oxygen tubing to the flow meter.

The nurse observes the unlicensed assistive personnel (UAP) securing a client's wrist restraints to the bed side rails. Which action is most important for the nurse to implement? a. Initiate the facility's restraint flow sheet. b. Demonstrate proper securing of the restraint. c. Ensure that the restraints are not too tight. d. Complete an adverse occurrence/incidence report.

b. Demonstrate proper securing of the restraint.

During the admission assessment of a terminally ill male client, The client states that he is an agnostic. What is the best nursing action in response to this statement. a. Provide information about the hours and location of the chapel. b. Document the statement in the client's spiritual assessment. c. Offer to contact a spiritual advisor of the client's choice. d. Invite the client to a healing service for people of all religions.

b. Document the statement in the client's spiritual assessment.

A client is discharged to a long-term care facility with an indwelling urinary catheter. Which nursing action should be included in the plan of care to reduce the client's risk of infection related to the catheter. a. Secure the drainage bag at bladder level during transport. b. Flush the catheter daily with sterile saline solution. c. Administer PRN Antipyretic if a fever develops. d. Encourage increased intake of oral fluids.

b. Flush the catheter daily with sterile saline solution.

A male hospice client with bone cancer reports to the nurse that his bone pain is not adequately controlled with his current dose of morphine sulfate, and he is experiencing difficulties with constipation. In addition to increasing the client's dose of laxative, what plan of treatment should the nurse anticipate? a. Reduce the dose of morphine b. Increase the dose of morphine c. Switch from morphine to codeine d. Take no additional morphine

b. Increase the dose of morphine

A client who has been taking diuretics for premenstrual swelling reports muscle weakness. Which serum electrolyte value should the nurse report to the healthcare provider. a. Total calcium 9.2 mg/dl (2.3 mmol/L) b. Potassium 3.1 mEq/L (3.1 mmol/L) c. Chloride 98 mEq/L (98 mmol/L) d. Sodium 142mEq/L (142 mmol/L)

b. Potassium 3.1 mEq/L (3.1 mmol/L) (LOW)

The nurse uses a sterile syringe to obtain a urine specimen, from a client's indwelling urinary catheter. After placing the specimen in a biohazard bag, the nurse transports the specimen to the lab. During what part of this procedure should the nurse wear gloves? a. Transporting the urine specimens to the laboratory. b. Removing the specimen from the catheter. c. Clamping the urinary catheter prior to the collection. d. Recording the output on the flowsheet in the client's room.

b. Removing the specimen from the catheter.

A female client's significant other has been at her bedside providing reassurances and support for the past 3days, as desired by the client. The client's estranged husband arrives and demands that the significant other not be allowed to visit or be given condition updates. Which intervention should the nurse implement? a. Obtain a perception from the healthcare provider regarding visitation privileges b. Request a consultation with the ethics committee for resolution of the situation c. Encourage the client to speak with her husband regarding his disruptive behavior d. Communicate the client's wishes to all members of the multidisciplinary team

b. Request a consultation with the ethics committee for resolution of the situation

1. The nurse notes that a client has cyanosis of the toes and fingertips. Which vital signs should the nurse obtain first. a. blood pressure. b. Respiratory rate. c. Pulse Rate. d. Temperature.

b. Respiratory rate. (Cyanosis is caused by low oxygen levels in the RBCs)

The nurse is teaching a husband how to care for his wife who recently had a stroke and has residual weakness on her right side. What style shoes does the nurse recommend the client wear when ambulating with her husband's assistance? a. Slip-on rubber shower shoes. b. Tennis shoes with Velcro. c. Rubber sole slippers. d. Leather sole loafers.

b. Tennis shoes with Velcro. (FALL PRECAUTION)

Two days after surgery, a male client experiences incisional pain while dangling his feet at the bedside and he refuses to ambulate as prescribed. The nurse establishes a problem of "activity intolerance related to pain". Which outcome statement is best for the nurse to include in the client's plan of care? a. To take analgesic as prescribed. b. To ambulate without discomfort. c. To show evidence of incision healing. d. To avoid pain-causing activity.

b. To ambulate without discomfort.

A client is admitted with complaints of shortness of breath (Dyspnea) on exertion, and chest pressure. The healthcare provider prescribes a medication that is unfamiliar to the nurse. When checking the drug handbook, the nurse reads that the prescribed amount is an unusually large dose. What actions should the nurse take? a. Consult pharmacists for those clarification. b. Verify the prescribed dosage with a healthcare provider. c. Administer the medication as prescribed. d. Give the dosage recommended in the drug handbook.

b. Verify the prescribed dosage with a healthcare provider.

What is the best approach for the nurse to use when interviewing a client about sexuality/ reproductive function? a. Ask questions in a vague, non-specific format b. Share personal values to put the client at ease c. Begin with questions that are less sensitive in nature d. Get the most difficult questions over with first

c. Begin with questions that are less sensitive in nature

A male client with a recent diagnosis of terminal cancer, tells his nurse that he wishes to die naturally. The client states that he's tired of fighting this illness and is only continuing treatment because of his family's wishes. What actions should the nurse take? a. Request a consultation for a psychologist to talk with the client. b. Call a clergy to discuss end-of-life decisions with the client. c. Determine if he wants to stop radiation and chemotherapy. d. Arrange a meeting with the client, his family and the healthcare provider.

c. Determine if he wants to stop radiation and chemotherapy.

The nurse is evaluating a client who is admitted to an adult medical unit, and notes that a client's urine output has been 70 ml/hr. Which action should the nurse implement? a. Recommend drinking cranberry juice with meals. b. Encourage the client to drink more fluids. c. Document the client's urinary output every hour. d. Notify the healthcare provider immediately.

c. Document the client's urinary output every hour. (NORMAL RANGE)

A male client with limited mobility is discharged with home-health services. When the home-health nurse arrives, the client asks what he can do for the swelling in his leg. What action should the nurse implement? a. Encourage the client to take short walks around the block. b. Advice the client to dangle his feet during meals and before bedtime. c. Ensure the clients to flex both of his feet, several times a day. d. Explain the need to keep the head of the bed elevated.

c. Ensure the clients to flex both of his feet, several times a day.

A policy requiring the removal of acrylic nails by all nursing personnel was implemented 6 months ago. Which assessment measure best determines if the intended outcome of the policy is being achieved. a. Number of staff induced injury b. Client satisfaction survey c. Health care-associated infection rate. d. Rate of needle-stick injuries by nurse.

c. Health care-associated infection rate.

An older male client returns to the clinic for chronic pain management after taking morphine sulphate (MS contin) 25mg every 12hrs. He states he took the medication only when the pain was too severe to sleep. What action should the nurse implement? a. Explain the risk of drug addiction from long term pain medication. b. Tell the client to continue taking the MS contin with severe pain. c. Instruct the client to take the MS Contin every 12 hours as prescribed. d. Teach the client alternative ways to manage his chronic pain.

c. Instruct the client to take the MS Contin every 12 hours as prescribed.

It is most important for the nurse to recalculate the Braden Scale score for a client who has developed which problem. a. Urinary incontinence. b. Hypo-active Bowel sound. c. Plus Two ankle Edema. d. Weakened cough efforts.

c. Plus Two ankle Edema.

When performing blood pressure measurement to assess for orthostatic hypotension, Which action should the nurse implement first? a. Apply the blood pressure cuffs securely. b. Assist the client to stand at the bedside. c. Position the client supine for a few minutes. d. Record the client's pulse rate and rhythm .

c. Position the client supine for a few minutes.

While interviewing a client, the nurse records the assessment in the electronic health record. Which statement is most accurate regarding electronic documentation during an interview? a. The interview process is enhanced with electronic documentation and allows the client to speak at a normal pace. b. Completing the electronic record during an interview is a legal obligation of the examining nurse. c. The nurse has limited ability to observe non-verbal communication while entering the assessment electronically. d. The client's comfort level is increased when the nurse breaks eye-contact to type notes into the record.

c. The nurse has limited ability to observe non-verbal communication while entering the assessment electronically.

What explanation is best for the nurse to provide a client who asked the purpose of using the log-rolling technique for turning? a. Working together can decrease the risk of back injury to the nurses. b. Turning instead of pulling reduces the likelihood of skin damage. c. The technique is intended to maintain straight spinal alignment. d. Using two or three people increases client's safety.

c. The technique is intended to maintain straight spinal alignment.

1. An adult client complains of insomnia and asks the nurse to recommend a sleeping pill. What reply is best for the nurse to provide? a. "Have you discussed this with your healthcare provider?" b. "Zolpidem Tartate (ambien) is used for insomnia." c. "Sleeping medication require side effects that require caution" d. "Tell me about your insomnia and how you treat it"

d. "Tell me about your insomnia and how you treat it"

A client newly diagnosed with stage 3 lung cancer becomes angry with the healthcare provider and nursing staff. Which intervention is most important for the nurse to implement? a. Arrange for the client to meet with another client who has lung cancer. b. Request a consultation from an oncology social worker. c. Acknowledge the client's anger and attempt to address its source. d. Allow the client and family time to be alone.

d. Allow the client and family time to be alone.

What self-care outcome is best for the nurse to use in evaluating a client's recovery from a stroke that resulted in left-sided hemiparesis? a. Self-care needs to be completed by the unlicensed assistive personnel. b. Participate in self-care to an optimal level of capacity. c. Promote independence by allowing clients to perform all self-care activities. d. Client verbalizes importance of hygienic practices in the recovery process.

d. Client verbalizes importance of hygienic practices in the recovery process.

A female nursing home resident and her family only speak Spanish. During a visit, the entire family begins to cry hysterically. When unable to determine why the family is upset, What intervention is most important for the nurse to implement? a. Sit quietly with the family to offer comfort and support. b. Ask a Spanish speaking staff member to talk with the family. c. Use a Spanish translation reference to interview the family. d. Close the door to the client's room to provide family privacy.

d. Close the door to the client's room to provide family privacy.

A male Native American presents to the clinic with complaints of frequent abdominal cramping and Nausea. He states that he has chronic constipation and has not had a bowel movement in 5 days, despite trying several home remedies. Which intervention is most important for the nurse to implement. a. Access for the presence of an impaction. b. Evaluate stool sample for the presence of blood. c. Obtain list of prescribed home medications. d. Determine what home remedies where used.

d. Determine what home remedies where used.

A nurse administers an opioid analgesic to a post operative client who also has severe obstructive sleep apnea (OSA). What intervention is most important for the nurse to implement before leaving the client alone? a. Apply the client's positive airway pressure device. b. Lift and lock the side rails in place. c. Remove dentures or other oral appliances. d. Elevate the head of the bed to 45 degree angle (d. Elevate the head of the bed to 45 degree angle

d. Elevate the head of the bed to 45 degree angle (INCREASED RISK FOR RESPIRATORY COMPLICATIONS)

A young male client with testicular cancer has a living will that describes his desire that no extraordinary measures be taken to save his life. The healthcare provider knows the client has a good prognosis and refuses to write a "Do Not Resuscitate"(DNR Prescription). What actions should the nurse take? a. Ensure resuscitation equipment is available. b. Ask the family to review options with the client. c. Place a DNR bracelet on the client's arm. d. Initiate an ethics committee review of the case.

d. Initiate an ethics committee review of the case.

The Practice Nurse (PN) applies sterile gloves and opens a pack of sterile sponges to assist the healthcare provider with a bedside procedure. After the Charge Nurse (CN) observes the PN, what actions should the charge nurse take? a. Confirm that PN is ready to assist with the planned procedure. b. Obtain all new supplies and directly assist with the procedure. c. Remove the contaminated package of sponges from the table. d. Instruct the PN to remove the gloves that are now contaminated.

d. Instruct the PN to remove the gloves that are now contaminated.

The nurse begins to suction a client's oropharynx as seen in the picture. What action should the nurse take next? a. Position suction in the trachea. b. Apply nasal cannula oxygen. c. Insert a tongue blade. d. Observe the suction secretion.

d. Observe the suction secretion.

In-home hospice care is arranged for a client with stage 4 lung cancer. While the palliative nurse is arranging for discharge, the client verbalizes concerns about pain. What action should the nurse implement? a. Explain the respiratory problems that can occur with morphine use. b. Teach family how to evaluate the effectiveness of analgesics. c. Recommend asking the healthcare professional for a patient-controlled analgesic (PCA) pump. d. Provide client with a schedule of around-the-clock prescribed analgesic use.

d. Provide client with a schedule of around-the-clock prescribed analgesic use.


Set pelajaran terkait

Pol 235 Final, Module 3 lessons learned points earned, POL 235 Lessens Learned Module 2

View Set

Pharmacology Prep U Chapter 25 Muscle Relaxants

View Set

ECH Chapter 7 Quiz, ECH Chapter 8 quiz, ECH Chapter 9 Quiz, ECH Chapter 10 Quiz, ECH Chapter 11 Quiz, ECH Chapter 6 Quiz, ECH Chapter 5 Quiz

View Set

Chapter 9- Skeletal Muscle Tissue

View Set

AP Review Questions for Chapter 39

View Set

Chapter 57: Management of Patients With Female Reproductive Disorders

View Set

Analyzing Data and Drawing Conclusions

View Set

The Financial Reporting Environment

View Set

Chapter 15 Alterations in Cognitive Systems

View Set